Mathcenter Forum

Mathcenter Forum (https://www.mathcenter.net/forum/index.php)
-   อสมการ (https://www.mathcenter.net/forum/forumdisplay.php?f=18)
-   -   อสมการ (https://www.mathcenter.net/forum/showthread.php?t=14949)

No.Name 25 ตุลาคม 2011 11:49

อสมการ
 
1. $a,b,c$ >0 $abc=1$

$$\dfrac{a+b+c}{3} \ge \sqrt[5]{\dfrac{a^2+b^2+c^2}{3}}$$

2. $a,b,c \ge 0$ ,$a^3+b^3+c^3=3$

$$a^4b^4+b^4c^4+c^4a^4 \le 3$$

3. $a,b,c >0$

$$\sqrt{\dfrac{a^3}{a^3+(b+c)^3}}+\sqrt{\dfrac{b^3}{b^3+(c+a)^3}}+\sqrt{\dfrac{c^3}{c^3+(a+b)^3}} \ge 1$$

จูกัดเหลียง 25 ตุลาคม 2011 18:29

อ้างอิง:

ข้อความเดิมเขียนโดยคุณ No.Name (ข้อความที่ 126534)
3. $a,b,c >0$

$$\sqrt{\dfrac{a^3}{a^3+(b+c)^3}}+\sqrt{\dfrac{b^3}{b^3+(c+a)^3}}+\sqrt{\dfrac{c^3}{c^3+(a+b)^3}} \ge 1$$

let $x=\frac{a}{b+c},y=\frac{b}{c+a},z=\frac{c}{a+b}$
Then It's Equavalent to $$\sqrt{\frac{x^3}{x^3+1}}+\sqrt{\frac{y^3}{y^3+1}}+\sqrt{\frac{z^3}{z^3+1}}\ge 1$$
use $f(x)=\frac{x^3}{x^3+1}$ then $f$ is convex by Jense's Ineuality
$$\sqrt{\frac{x^3}{x^3+1}}+\sqrt{\frac{y^3}{y^3+1}}+\sqrt{\frac{z^3}{z^3+1}} \ge 3\sqrt{\frac{(\frac{x+y+z}{3})^3}{(\frac{x+y+z}{3})^3+1}}$$
Then It's Remain to show that $x+y+z\ge \frac{3}{2}$
That's Nesbitt's Ineqaulity

No.Name 25 ตุลาคม 2011 19:35

$4.a,b,c >0, a^2+b^2+c^2+d^2=4$

$$a^3+b^3+c^3+d^3 \le 8$$


$5. a,b,c >0 such that$

$$a^2+b^2+c^2=a+b+c$$

prove that

$$a^2b^2+b^2c^2+c^2a^2 \le ab+bc+ca$$

PP_nine 25 ตุลาคม 2011 20:00

อ้างอิง:

ข้อความเดิมเขียนโดยคุณ No.Name (ข้อความที่ 126543)
$4.a,b,c >0, a^2+b^2+c^2+d^2=4$

$$a^3+b^3+c^3+d^3 \le 8$$


อสมการนี้ยังไม่ sharp ครับ ก็คือมันเป็นไปไม่ได้ที่ $a^3+b^3+c^3+d^3 = 8$

เพราะ $a,b,c,d>0$ แล้วทำให้ $a^2<4$ เสมอ

PP_nine 25 ตุลาคม 2011 20:16

อ้างอิง:

ข้อความเดิมเขียนโดยคุณ No.Name (ข้อความที่ 126543)
$5. a,b,c >0 such that$

$$a^2+b^2+c^2=a+b+c$$

prove that

$$a^2b^2+b^2c^2+c^2a^2 \le ab+bc+ca$$

จาก$$a^2+b^2+c^2=a+b+c$$ยกกำลังสองได้$$a^4+b^4+c^4+2(a^2b^2+b^2c^2+c^2a^2)=a^2+b^2+c^2+2(ab+bc+ca)$$จึงเป็นการเพียงพอที่จะพิสูจน ์ว่า$$a^2+b^2+c^2 \le a^4+b^4+c^4$$ซึ่งสมมูลกับ$$(a^2+b^2+c^2)^3 \le (a^4+b^4+c^4)(a+b+c)^2$$หรือก็คือ$$a^2+b^2+c^2 \le \sqrt[3]{(a^4+b^4+c^4)(a+b+c)(a+b+c)}$$ซึ่งจริงโดย Holder

No.Name 25 ตุลาคม 2011 20:22

อ้างอิง:

ข้อความเดิมเขียนโดยคุณ PP_nine (ข้อความที่ 126544)
อสมการนี้ยังไม่ sharp ครับ ก็คือมันเป็นไปไม่ได้ที่ $a^3+b^3+c^3+d^3 = 8$

เพราะ $a,b,c,d>0$ แล้วทำให้ $a^2<4$ เสมอ

ค่าสูงสุดของมันเป็นได้ที่ 4 ไม่ใช่หรอครับ

ปล. Holder มันใช้ยังไงหรอครับ

Keehlzver 25 ตุลาคม 2011 20:24

Hint หรือ เฉลย หรือเอามาให้โพส์ทำเฉยๆครับ?

ข้อ 1 ของ vasc ผมเคยเฉลยไปแล้วใช้ sos-schur อีกวิธีที่ทำได้คือ pqr (ยังไม่ได้ลอง แต่มั่นใจว่าน่าจะทำได้) หรือไม่ก็น่าจะมีวิธีพื้นฐานดีๆ

ข้อ 2 อาจจะอยู่ใน secret หรือเปล่า

No.Name 25 ตุลาคม 2011 20:31

อ้างอิง:

ข้อความเดิมเขียนโดยคุณ Keehlzver (ข้อความที่ 126547)
Hint หรือ เฉลย หรือเอามาให้โพส์ทำเฉยๆครับ?

ข้อ 1 ของ vasc ผมเคยเฉลยไปแล้วใช้ sos-schur อีกวิธีที่ทำได้คือ pqr (ยังไม่ได้ลอง แต่มั่นใจว่าน่าจะทำได้) หรือไม่ก็น่าจะมีวิธีพื้นฐานดีๆ

ข้อ 2 อาจจะอยู่ใน secret หรือเปล่า

เอามาให้ทำน่ะครับ เพราะผมก็ัยังทำไม่ได้ข้อไหนที่ผมไหนที่ผมได้ก็จะลงวิธีทำไว้

เพื่อจะให้คนมาดูว่าถูกหรือผิด

PP_nine 25 ตุลาคม 2011 20:51

อ้างอิง:

ข้อความเดิมเขียนโดยคุณ No.Name (ข้อความที่ 126546)
ค่าสูงสุดของมันเป็นได้ที่ 4 ไม่ใช่หรอครับ

ปล. Holder มันใช้ยังไงหรอครับ

ก็ถ้าเป็น $a^2=4$ ดูสิครับ แทนในสมการเดิมจะกลายเป็นว่า $b^2+c^2+d^2=0$

แต่ $b,c,d \in \mathbb{R}$ ก็แสดงว่า $b=c=d=0$ กรณีเดียว ซึ่งขัดกับที่ว่า $a,b,c,d>0$ ยังไงล่ะครับ :)

ส่วนเรื่องของ Holder ที่ผมทำอาจจะเร่งรีบจนลัดไปหน่อย :p

อสมการโฮลเดอร์ก็มีอยู่ว่า $$x_1y_1+x_2y_2+...+x_ny_n \le \left(\, x_1^p+x_2^p+...+x_n^p\right)^{1/p} \left(\, y_1^q+y_2^q+...+y_n^q\right)^{1/q}$$ เมื่อทุกตัวแปรเป็นจำนวนจริงบวก และ $1/p+1/q=1$ ซึ่งก็เป็นทฤษฎีใหญ่ที่รวมเอาของ Cauchy-Schwarz เข้ามาด้วย

แถมสามารถเพิ่มเป็นกี่ตัวแปรก็ได้ ไม่จำเป็นต้องมีแค่ $x_i,y_i$ ถ้าเป็นสามตัวแปรก็คือ $$x_1y_1z_1+x_2y_2z_2+...+x_ny_nz_n \le \left(\, x_1^p+x_2^p+...+x_n^p\right)^{1/p} \left(\, y_1^q+y_2^q+...+y_n^q\right)^{1/q} \left(\, z_1^r+z_2^r+...+z_n^r\right)^{1/r}$$ เมื่อทุกตัวแปรเป็นจำนวนจริงบวก และ $1/p+1/q+1/r=1$

ตัวที่ใช้บ่อยก็คือ $p=q=r=3$ และ $n=3$ หรือก็คือ $$x_1y_1z_1+x_2y_2z_2+x_3y_3z_3 \le \sqrt[3]{(x_1^3+x_2^3+x_3^3)(y_1^3+y_2^3+y_3^3)(z_1^3+z_2^3+z_3^3)}$$ และในตัวอย่างที่ผมให้ไปก็คือ $$x^2+y^2+z^2=x^{4/3}x^{1/3}x^{1/3}+y^{4/3}y^{1/3}y^{1/3}+z^{4/3}z^{1/3}z^{1/3} \le \sqrt[3]{(x^4+y^4+z^4)(x+y+z)(x+y+z)}$$

No.Name 25 ตุลาคม 2011 21:53

อ้างอิง:

ข้อความเดิมเขียนโดยคุณ PP_nine (ข้อความที่ 126551)
ก็ถ้าเป็น $a^2=4$ ดูสิครับ แทนในสมการเดิมจะกลายเป็นว่า $b^2+c^2+d^2=0$

แต่ $b,c,d \in \mathbb{R}$ ก็แสดงว่า $b=c=d=0$ กรณีเดียว ซึ่งขัดกับที่ว่า $a,b,c,d>0$ ยังไงล่ะครับ :)

ส่วนเรื่องของ Holder ที่ผมทำอาจจะเร่งรีบจนลัดไปหน่อย :p

อสมการโฮลเดอร์ก็มีอยู่ว่า $$x_1y_1+x_2y_2+...+x_ny_n \le \left(\, x_1^p+x_2^p+...+x_n^p\right)^{1/p} \left(\, y_1^q+y_2^q+...+y_n^q\right)^{1/q}$$ เมื่อทุกตัวแปรเป็นจำนวนจริงบวก และ $1/p+1/q=1$ ซึ่งก็เป็นทฤษฎีใหญ่ที่รวมเอาของ Cauchy-Schwarz เข้ามาด้วย

แถมสามารถเพิ่มเป็นกี่ตัวแปรก็ได้ ไม่จำเป็นต้องมีแค่ $x_i,y_i$ ถ้าเป็นสามตัวแปรก็คือ $$x_1y_1z_1+x_2y_2z_2+...+x_ny_nz_n \le \left(\, x_1^p+x_2^p+...+x_n^p\right)^{1/p} \left(\, y_1^q+y_2^q+...+y_n^q\right)^{1/q} \left(\, z_1^r+z_2^r+...+z_n^r\right)^{1/r}$$ เมื่อทุกตัวแปรเป็นจำนวนจริงบวก และ $1/p+1/q+1/r=1$

ตัวที่ใช้บ่อยก็คือ $p=q=r=3$ และ $n=3$ หรือก็คือ $$x_1y_1z_1+x_2y_2z_2+x_3y_3z_3 \le \sqrt[3]{(x_1^3+x_2^3+x_3^3)(y_1^3+y_2^3+y_3^3)(z_1^3+z_2^3+z_3^3)}$$ และในตัวอย่างที่ผมให้ไปก็คือ $$x^2+y^2+z^2=x^{4/3}x^{1/3}x^{1/3}+y^{4/3}y^{1/3}y^{1/3}+z^{4/3}z^{1/3}z^{1/3} \le \sqrt[3]{(x^4+y^4+z^4)(x+y+z)(x+y+z)}$$

ขอบคุณมากครับ มีตัวอย่างโจทย์อสมการโฮลเดอร์ บ้างไหมครับ

ลองไปหาดูในเน็ตเจอแต่ยากๆอ่ะครับ เจอข้อนึงไม่รู้ถูกไหมนะครับตรวจให้ด้วย

$ab^2+bc^2+ca^2\leq a^3+b^3+c^3$

$ab^2+bc^2+ca^2=a \cdot b \cdot b+b\cdot c \cdot c+c\cdot a \cdot a$

$ \cdot b \cdot b+b\cdot c \cdot c+c\cdot a \cdot a \le \sqrt[3]{(a^3+b^3+c^3)^3}$

$ab^2+bc^2+ca^2\leq a^3+b^3+c^3$

PP_nine 25 ตุลาคม 2011 22:15

อ้างอิง:

ข้อความเดิมเขียนโดยคุณ No.Name (ข้อความที่ 126556)
ขอบคุณมากครับ มีตัวอย่างโจทย์อสมการโฮลเดอร์ บ้างไหมครับ

ลองไปหาดูในเน็ตเจอแต่ยากๆอ่ะครับ เจอข้อนึงไม่รู้ถูกไหมนะครับตรวจให้ด้วย

$ab^2+bc^2+ca^2\leq a^3+b^3+c^3$

$ab^2+bc^2+ca^2=a \cdot b \cdot b+b\cdot c \cdot c+c\cdot a \cdot a$

$a \cdot b \cdot b+b\cdot c \cdot c+c\cdot a \cdot a \le \sqrt[3]{(a^3+b^3+c^3)^3}$

$ab^2+bc^2+ca^2\leq a^3+b^3+c^3$

ถูกแล้วครับๆ :great:

ส่วนโจทย์ลองหาดูในลิ้งค์ที่ผมเคยโพสไปแล้วในนี้ หนังสือเข้าค่ายสอวน

รู้สึกจะเป็นเล่มที่ผมเขียนต่อท้ายว่า อันนี้ดีๆ

ตัวอย่างโจทย์ซักข้อที่ดูน่าจะยาก แต่ถ้าใช้ Holder เป็นก็จะมองออกว่าทำอย่างไร

กำหนดจำนวนจริง $a,b,c>0$ พิสูจน์ $$\frac{a+\sqrt{ab}+\sqrt[3]{abc}}{3} \le \sqrt[3]{a \cdot \frac{a+b}{2} \cdot \frac{a+b+c}{3}}$$

ปล.ไม่ได้ใช้ Holder อย่างเดียวนะ กลัวเดี๋ยวออกทะเล :laugh:

template 25 ตุลาคม 2011 22:31

5.\[\sum_{cyc}a^2b^2=\frac{(\sum_{cyc}a)^2\sum_{cyc}a^2b^2}{(\sum_{cyc}a^2)^2}=\frac{\sum_{sym}a^4b^2+2\sum_{cyc}a^3b^3+2\sum_{sym}a ^3b^2c+3a^2b^2c^2}{(\sum_{cyc}a^2)^2}\]
\[\le\frac{\sum_{sym}a^5b+\sum_{cyc}a^4bc+2\sum_{cyc}a^3b^3+2\sum_{sym}a^3b^2c}{(\sum_{cyc}a^2)^2}=\frac{(\sum_{cyc}a^2)^2\sum_{cy c}ab}{(\sum_{cyc}a^2)^2}=\sum_{cyc}ab\]

Keehlzver 25 ตุลาคม 2011 23:49

มีกระทู้เก่าๆ เป็นโจทย์พี่ nooonuii ครับ http://www.mathcenter.net/forum/showthread.php?t=10634 มีอสมการโฮลเดอร์ให้ฝึกเยอะมาก :great:

ปล.คุณ template ลองทำแบบไม่กระจายดูซิครับ :rolleyes:

No.Name 27 ตุลาคม 2011 18:09

อ้างอิง:

ข้อความเดิมเขียนโดยคุณ PP_nine (ข้อความที่ 126558)
ถูกแล้วครับๆ :great:

ส่วนโจทย์ลองหาดูในลิ้งค์ที่ผมเคยโพสไปแล้วในนี้ หนังสือเข้าค่ายสอวน

รู้สึกจะเป็นเล่มที่ผมเขียนต่อท้ายว่า อันนี้ดีๆ

ตัวอย่างโจทย์ซักข้อที่ดูน่าจะยาก แต่ถ้าใช้ Holder เป็นก็จะมองออกว่าทำอย่างไร

กำหนดจำนวนจริง $a,b,c>0$ พิสูจน์ $$\frac{a+\sqrt{ab}+\sqrt[3]{abc}}{3} \le \sqrt[3]{a \cdot \frac{a+b}{2} \cdot \frac{a+b+c}{3}}$$

ปล.ไม่ได้ใช้ Holder อย่างเดียวนะ กลัวเดี๋ยวออกทะเล :laugh:

ผมได้เพียงว่า

$\dfrac{a+\dfrac{a+b}{2}+\dfrac{a+b+c}{3}}{3} \ge \sqrt[3]{a \cdot \dfrac{a+b}{2} \cdot \dfrac{a+b+c}{3}}$

$\dfrac{a+\dfrac{a+b}{2}+\dfrac{a+b+c}{3}}{3} \ge \dfrac{a+\sqrt{ab}+\sqrt[3]{abc}}{3}$

มันไม่เีพียงพอที่จะพิสูจน์ อสมการที่ต้องการอ่ะครับ (หรือหลงป่า ตั้งแต่เริ่มเลย)

จูกัดเหลียง 27 ตุลาคม 2011 20:14

อ้างอิง:

ข้อความเดิมเขียนโดยคุณ PP_nine (ข้อความที่ 126558)
ตัวอย่างโจทย์ซักข้อที่ดูน่าจะยาก แต่ถ้าใช้ Holder เป็นก็จะมองออกว่าทำอย่างไร

กำหนดจำนวนจริง $a,b,c>0$ พิสูจน์ $$\frac{a+\sqrt{ab}+\sqrt[3]{abc}}{3} \le \sqrt[3]{a \cdot \frac{a+b}{2} \cdot \frac{a+b+c}{3}}$$

$$\frac{a+\sqrt{ab}+\sqrt[3]{abc}}{3}=\frac{a^{\frac{1}{3}}\cdot a^{\frac{1}{3}}\cdot a^{\frac{1}{3}}+a^{\frac{1}{3}}\cdot b^{\frac{1}{3}}\cdot (\sqrt{ab})^{\frac{1}{3}}+a^{\frac{1}{3}}\cdot b^{\frac{1}{3}}\cdot c^{\frac{1}{3}}}{3} $$
$$\leq \frac{1}{3}\sqrt[3]{((a^{\frac{1}{3}})^3+(a^{\frac{1}{3}})^3+(a^{\frac{1}{3}})^3)(a+b+\sqrt{ab})(a+b+c)}$$
$$\leq \frac{1}{3}\sqrt[3]{((a^{\frac{1}{3}})^3+(a^{\frac{1}{3}})^3+(a^{\frac{1}{3}})^3)(\frac{3}{2}(a+b))(a+b+c)}$$ $$=\sqrt[3]{a \cdot \frac{a+b}{2} \cdot \frac{a+b+c}{3}}$$


เวลาที่แสดงทั้งหมด เป็นเวลาที่ประเทศไทย (GMT +7) ขณะนี้เป็นเวลา 20:43

Powered by vBulletin® Copyright ©2000 - 2024, Jelsoft Enterprises Ltd.
Modified by Jetsada Karnpracha